[obm-l] Re: [obm-l] Re: [obm-l] Problema de lógica (impossivel para mim)

2002-04-22 Por tôpico Ralph Teixeira

Bati sem querer em enviar... ainda falta MUITO para resolver o
problema... Continuo onde parei, já trocando (VI) e (VII) por (IX) como
disse antes. Para que ninguém fique decepcionado -- no final, NÃO HÁ SOLUÇÂO
(ou eu errei alguma coisa).


(I)  ?  Hart IR   

(II) ? Stev   
  (possiveis linhas aqui...)
 ?   RL   

(III) Larry
   Korn
GD
 Par
(ordem das linhas aqui pode mudar)

(IV) ?  Rose  
 ?    
 ?    31

(V) 3    (=x-10 ou =x+10) onde x eh definido assim:? 
Baird  x

(VII) ?   GP 
  ?    2a

(IX) ? Bert   onde ? Tom   
 ?    y+15 ?    y
     d-8

Bom, note que como Bert, Tom e Steve ficaram na frente de alguém, então
Larry ficou em 4o. Note que Bert e Mr. Rose ficaram duas posições na frente
de alguém, então eles estão em 1o ou 2o. Há assim quatro casos a considerar:

(i) Bert em 1o, Mr. Rose em 2o. Inserimos (IV) e (IX) na nossa tabela assim:

1 Bert   
2  Rose  
3    y+15
4 Larr   d-8=31

Então d=39 é o maior número. Mas então a parte da direita de (IX) não tem
onde entrar (Tom em 1o conflita com Bart; Tom em segundo faz y=y+15; Tom em
terceiro faz y=31, y+15=46d=39).

(ii) Bert em 2o, Mr. Rose em 1o. Inserimos (IV) e (IX) na tabela assim:

1  Rose  
2 Bert   
3    y+15=31
4 Larr   d-8

Agora, y=16. De novo, Tom em 2o conflita com Bert; Tom em 3o (por (IX)) faz
y=d-8 e então y+15=31d=24, dá pau. Concluímos que Tom ficou em 1o, e Steve
fica com o 3o; como Steve está acima do Ruy Lopez (II), completamos para:

1 Tom  Rose  
2 Bert   16
3 Stev   31
4 Larr  RL   d-8

Quem está em 4o? Não é Hart (que usou abertura IR, por (I)), nem Korn (regra
(III)), então é Baird:

1 Tom  Rose  
2 Bert   16
3 Stev   31
4 Larr Bair  d-8

onde d-8=31+10 ou d-8=31-10, isto é, d=49 ou d=29. Esta última não pode
ser (d é o maior número!). De qualquer forma, d=49 ou 50 faz d-8=41 ou 42 ,
e os 4 números têm de ser {d,d-8,31,16} com o menor a=16 -- mas (VII) diz
que 2a=32 tinha de aparecer. Deu pau.

(iii) Bert Rose ficou em 1o:

1 Bert Rose  
2    y+15
3    d-8=31
4 Larr   

E d=39 é o maior número. Tom é o 3o então (IX) (em 2o dava pau pois y=31 faz
y+15=46d), e Steve sobra para 2o.

1 Bert Rose  
2 Stev   y+15
3 Tom    d-8=31
4 Larr   y

Agora, por (V) Baird (na colocação 2 ou 4) deve estar a 10 ou mais unidades
de 31; como d=39 é o maior, a mesa de Baird é 21 ou menos, portanto y=21 e
nem y nem y+15 podem ser d=39. Então, d=39 é a mesa do 1o colocado, y=a é o
menor; 2a deve ser então y+15. Portanto, a=y=15, y+15=30 e ficamos com:

1 Bert Rose  d=39
2 Stev   y+15=2a=30
3 Tom    d-8=31
4 Larr   y=a=15

Pela regra (V), Baird está em 4o. Por (VII), Bert abriu GP. Mas Bert Rose
que ficou em 1o não é nenhum dos 4 jogadores mencionados em (III) deu
pau de novo!

1 Bert Rose GP   d=39
2 Stev   y+15=2a=30
3 Tom    d-8=31
4 Larr Bair  y=a=15

(iv) Enfim, Bert Rose ficou em 2o lugar!

1    
2 Bert Rose  
3    y+15
4 Larr   d-8=31

O papo é parecido com o de (iii). d=39 é o maior, Tom deve estar em 1o por
(IX) e Steve fica em 3o. Como bonus, por (II) colocamos a Ruy Lopez em 4o.

1 Tom    
2 Bert Rose  y
3 Stev   y+15
4 Larr  RL   d-8=31

Onde fica d=39? Argh, abra em dois casos de novo:

(a) d=39 em 1o. Então y=a é o menor, e 2a tem de ser y+15, e os números são
30,15,30,31 como no fim do caso (iii):

1 Tom    d=39
2 Bert Rose  y=a=15
3 Stev   y+15=2a=30
4 Larr  RL   d-8=31

Use (VII) para colocar GP para Bert Rose, mas deu pau de novo, do mesmo
jeito que antes: Bert Rose não é nenhum dos jogadores mencionados em (III).

(b) Bom, enfim, tentamos d=39=y+15, e y=24.

1 Tom    
2 Bert Rose  y=24
3 Stev   y+15=d=39
4 Larr  RL   d-8=31

A regra (VII) força que GP seja a abertura de Tom, e 2a=24 faz a=12:

1 Tom   GP   a=12
2 Bert Rose  y=2a=24
3 Stev   y+15=d=39
4 Larr  RL   d-8=31

Enfim, Baird deve ser Tom (a mais de 10 unidades de 39), Hart com IR vai em
3o, e Korn sobra para Larr:


1 Tom  Bair GP   a=12
2 Bert Rose GD   y=2a=24
3 Stev Hart IR   y+15=d=39
4 Larr Korn RL   d-8=31

E de novo deu pau, pois Steve Hart não foi mencionado em (III).

Então, não há solução E eu acabei de perder este tempo todo digitando
isso... Argh... Bolas Droga... *@¨($¨@(%...

:)

Abraço,
Ralph

=
Instruções para entrar na lista, sair da lista e usar a 

Re: [obm-l] 1,0000...001

2002-04-22 Por tôpico Ralph Teixeira

Hmmm Eu já escrevi um bocado sobre o assunto 0,...=1; eu lembro
de ter focado mais na questão que o Rafael levanta aqui, isto é, ao invés de
provar que 0,999...=1, eu dei a minha opinião do porquê da SURPRESA e da
resistência que as pessoas têm de aceitar este fato.

Mas vou parafrasear o Nicolau: está nos arquivos da lista, eu tive o
maior trabalho para escrever com cuidado da outra vez, então quem quiser
veja lá. Afinal, é para isso que servem os arquivos. ;) ;) ;)

Mas o resumo é: sim, Rafael, a maioria das pessoas têm dificuldade de
aceitar a idéia que um número possa ter duas representações decimais
distintas, pois elas aprenderam a ver se dois números são iguais verificando
se a expansão decimal é a mesma. Todos usavam as expansões para comparar
números, era a maneira mais simples de ver qual é maior, e de repente vem
alguém e diz que essa regra de comparação não funciona. Pior, não funciona
num caso específico (da dízima com ...)... então a reação natural é
este caso está errado ao invés de minha regra estava errada, falha apenas
neste caso. Quanto aos casos que você mencionou, as pessoas pensam que 0
não é nada, então não conta -- para ver qual é maior, o 0 não contava de
qualquer jeito. Mas seu argumento é ótimo.

Tudo isso, é claro, em minha modesta opinião (EMMO?).

Abraço,
Ralph

 - Original Message -
From: Rafael WC [EMAIL PROTECTED]
To: [EMAIL PROTECTED]
Sent: Monday, April 22, 2002 4:40 PM
Subject: Re: [obm-l] 1,...001


Concordo com Fred sobre minha questão do 1,000...001,
era justamente dessa maneira que tinha pensado. O que
faria com que assustássemos a todos com:
0,999... = 1 = 1,000...001

Eu estive pensando por que é que incomoda tanto
0,999... = 1??
É por ter duas grafias decimais para o mesmo número?
Então porque é que ninguém questiona os valores
desses:
1,000...
001
1,00
001,000

Não é tudo 1? E cada um não escrevo de um jeito? Por
que é que os noves incomodam mais que os zeros???

Rafael.


=
Instruções para entrar na lista, sair da lista e usar a lista em
http://www.mat.puc-rio.br/~nicolau/olimp/obm-l.html
O administrador desta lista é [EMAIL PROTECTED]
=



RE: [obm-l] Problema Interessante!

2002-04-20 Por tôpico Ralph Teixeira

 
Solucoes todas otimas, rapidas e diretas.

Mas... se voce quiser outro jeito... ok:

 10x - 2xy + y = 0 

Ponha o 2x em evidencia...
2x(5-y)+y=0

Junte uma constante para criar um multiplo do 5-y (no caso, -1(5-y))...
2x(5-y)-(5-y)=-5
(2x-1)(5-y)=-5
(2x-1)(y-5)=5

Entao 2x-1 e y-5 sao divisores de 5... Como 2x-10, tem-se
(2x-1,y-5) = (1,5) ou (5,1). O primeiro dah (x,y)=(1,10) (nao pode), o
segundo dah (x,y)=(3,6) (OK!).

(Veja bem, esta solucao fica PIOR do que as outras, pois hah poucos casos a
considerar... Mas eu queria fazer este comentario para dar uma dica quando o
pessoal enfrentar coisas parecidas onde hah MUITAS opcoes para x e y no
comeco do problema... Com este metodo, voce acharia *todas* as solucoes
inteiras da equacao dada.)

Abraco,
  Ralph

=
Instruções para entrar na lista, sair da lista e usar a lista em
http://www.mat.puc-rio.br/~nicolau/olimp/obm-l.html
O administrador desta lista é [EMAIL PROTECTED]
=



RE: [obm-l] Limites

2002-04-13 Por tôpico Ralph Teixeira

 
H.. esta questao tem algo cheirando a armadilha

Veja bem, a questao nao deixa claro se a gente tah falando de uma funcao ou
uma sequencia. Se for funcao, eu concordo com o Carlos. Mas se for
sequencia, isto eh, soh para n inteiro, a coisa muda. Afinal, note que

lim (n-oo) sqrt(n^2+n+1)-n = 1/2

(Sai multiplicando pelo conjugado e fazendo as contas). Assim,

cos (pi*sqrt(n^2+n+1))= +- cos(pi*sqrt(n^2+n+1)-pi*n)

enquanto (o sinal + ou - depende se n eh par ou impar)

cos(pi*(sqrt(n^2+n+1)-n)) tende a cos(pi/2)=0 (!?!)

O denominador vai para 1 mesmo... Assim, o limite da SEQUENCIA eh 0.

(Mas repito, se n for uma variavel REAL, o que o Carlos disse estah
perfeito).

Abraco,
 Ralph


-Original Message-
From: Carlos Victor
To: [EMAIL PROTECTED]
Sent: 4/12/02 7:27 PM
Subject: Re: [obm-l] Limites


Olá  Carol ,
Se é  realmente  o que  entendi , faça  o seguinte : olhe  para a 
expressão  (n^3 - n + 1)^1/n  e a coloque  da seguinte forma n^(3/n) 
sqrt(1-1/n^2 +1/n^3)^(1/n) .Observe  que  esta  expressão 
tem  limite  igual  a 1  e que a expressão  em coseno  fica  oscilando 
entre  -1  e 1 ; portanto  o limite  não  existe , ok ?

Abraços  , Carlos  Victor


At 05:47 11/4/2002 -0300, Ana Carolina Boero wrote:
Por favor, como calculo este limite?

lim   { cos [pi * sqrt(n^2 + n + 1)] } / (n^3 - n + 1)^1/n
n-oo

Muito obrigada!

Carol
=
Instruções para entrar na lista, sair da lista e usar a lista em
http://www.mat.puc-rio.br/~nicolau/olimp/obm-l.html
O administrador desta lista é [EMAIL PROTECTED]
=



[obm-l] Re: [obm-l] álgebra II

2002-04-04 Por tôpico Ralph Teixeira

Sejam a,b,c,d inteiros positivos tais que a^5 = b^4,
c³ = d² e c - a = 19. Determine o valor de d - b.

H Vejamos.

Note que a^5=b^4 tem de ser uma 20a potencia perfeita, isto eh,
a^5=b^4=m^20.
Assim, a=m^4  e b=m^5.

Também, c^3=d^2 tem de ser uma 6a potencia perfeita, isto eh, c^3=d^2=n^6.
Assim, c=n^2 e d=n^3.

Isto quer dizer que c-a = n^2-m^4=(n-m^2)(n+m^2)=19.

Mas 19 é primo, então n-m^2=1 e n+m^2=19. Resolva, ache n e m, entao voce
sabe a,b,c e d.

Abraço,
Ralph

P.S.: Vejo agora que minha solução é equivalente à do Arnaldo... mas, de
qualquer forma, eu prefiro este jeito de escrevê-la. :)

=
Instruções para entrar na lista, sair da lista e usar a lista em
http://www.mat.puc-rio.br/~nicolau/olimp/obm-l.html
O administrador desta lista é [EMAIL PROTECTED]
=



[obm-l] RE: [obm-l] Re: [obm-l] funções e poliminós

2002-03-26 Por tôpico Ralph Teixeira

 2.Determine todas as funções estritamente crescentes f:N-N tais que 
f(n+f(n))=2f(n) 

Interessante A resposta é múltipla:

i) Qualquer função do tipo f(n)=n+a para a=0 fixo;
ii) Ou qualquer função do tipo f(0)=0 e f(n)=n+a para n0, com a=0 fixo.

Em primeiro lugar note que, se f é estritamente crescente, então
f(n+1)=f(n)+1, e a igualdade só ocorre se f(n) e f(n+1) forem consecutivos,
certo? Ora, assim, f(a+b)=f(a+b-1)+1=f(a+b-2)+2=...=f(a)+b para
quaisquer a,b naturais. A igualdade (f(a+b)=f(a)+b) só ocorre se
f(a),f(a+1), f(a+2),...,f(a+b) forem números consecutivos!

Em particular, f(n+f(n))=f(n)+f(n)=2f(n); a igualdade só ocorre se f(n),
f(n+1), f(n+2), ..., f(n+f(n)) forem todos consecutivos!

(Fazendo um pequeno aparte, note que se f é estritamente crescente, então
f(n)=0 só pode ter, no máximo, n=0 como solução.)

Como a igualdade OCORRE, olhe os dois primeiros termos da lista:
f(n+1)=f(n)+1 para qualquer n (exceto possivelmente se f(n)=0, quando a
nossa lista acima só tem f(n) e nem chega até f(n+1), de qualquer forma,
isto só pode ocorrer se n=0, né?).

Agora é fácil: como f(n+1)=f(n)+1 para qualquer n (exceto 0), tem-se
f(n)=n+a para qualquer n (onde a é fixo; a=f(1)-1, digamos).

Verifiquemos se precisa de mais algo: tomando f(n)=n+a, tem-se

f(n+f(n))=f(n+(n+a))=f(2n+a)=(2n+a)+a=2n+2a=2f(n) (OK!)

Enfim, para n=0 temos:

i) Se f(0)=0, então f(0+f(0))=f(0+0)=f(0)=0, e 2f(0)=0; funciona ok.
ii) Se f(0)=x0, então f(0+f(0))=f(0+x)=x+a, e 2f(0)=2x; conclui-se que
x+a=2x, isto é, f(0)=a. Pode ser também!

Resposta final:

f(n)=n+a para a fixo e qualquer n0
f(0)=0 OU f(0)=a (escolha o que você quiser).

3. É possível empacotar 250 tijolos 1x1x4 em uma caixa de dimensões 
10x10x10?

 H Nao.

 Sejam (i,j,k) os índices de cada um dos cubinhos 1x1x1 da caixa
10x10x10 (0=i,j,k=9).
 Sejam A0={cubinhos onde i+j+k=0 mod 4}, A1={cubinhos onde i+j+k=1 mod
4}, A2={=2 mod4} e A3={=3 mod 4}. Em outras palavras, An é o
conjunto dos cubinhos cujos índices i+j+k deixam resto n na divisão por 4.

 Bom, primeiro convença-se de que cada tijolo 1x1x4 necessariamente
ocupará um cubinho de cada tipo (A0, A1, A2, A3). Assim, se fosse possível
cobrir a caia 10x10x10, teríamos 250 cubinhos de cada tipo.

 Mas quantos cubinhos do tipo A0 existem? Bom, a resposta é: há tantos
cubinhos quantas forem as soluções de i+j+k=4m com 0=i,j,k=9. Olhando só
os restos na divisão por 4, temos as seguintes opções para i, j e k:
{0,1,2,3,0,1,2,3,0,1}. Veja os casos (tudo é feito modulo 4):

i) i=0 (3 opções) então j+k=0; temos j=0,k=0 (3x3 opções) ou j=1,k=3 (3x2
opções) ou j=k=2 (2x2 opções) ou j=3,k=1 (2x3 opções).
TOTAL DE SOLUÇÕES AQUI: 3.(9+6+4+6) = 75

ii) i=1 (3 opções) então j+k=3; as opções para (j,k) são (0,3), (1,2), (2,1)
ou (3,0) com 3x2+3x2+2x3+2x3=24 opções para j e k.
TOTAL: 3x24=72 opções aqui.

iii) i=2 (2 opções) então j+k=2; (j,k)=(0,2),(1,1),(2,0) ou (3,3). Total:
2x(3x2+3x3+2x3+2x2)=50 opções

iv) i=3 (2 opções) e j+k=1; (j,k)=(0,1),(1,0),(2,3)ou(3,2) com
2x(3x3+3x3+2x2+2x2)=52 opções.

Ou seja, há um total de 75+72+50+52=249 cubinhos do tipo A0. Mas, para
cobrir o cubo 10x10x10 com aqueles tijolinhos, teríamos de cobrir 250
cubinhos do tipo A0! Absurdo, portanto não é possível cobri-los.

--//--

Você podia contar cubinhos no braço também... Fica mais fácil de ver numa
figura, mas a idéia é que os cubinhos A0 são dos tipos:
 i+j+k=0 (o cubinho do canto)
 i+j+k=4 (um plano de cubinhos; use combinatória ou conte mesmo os
cubinhos aqui caso a caso: i=0 implica j+k=4, com 5 soluções; i=1 implica
j+k=3, com 4 soluções;... etc etc TOTAL: 5+4+3+2+1=15 cubinhos)
 i+j+k=8 (outro plano de cubinhos, com 9+8+7+6+...+1=45 cubinhos)
 i+j+k=12 (7+8+9+10+9+8+7+6+5+4=73 cubinhos)
 i+j+k=16 (3+4+5+6+7+8+9+10+9+8=69 cubinhos)
 i+j+k=20 (1+2+3+4+5+6+7+8=45 cubinhos)
 i+j+k=24 (1+2+3+4=10 cubinhos)
 i+j+k=28 (não dá mais)

Em suma, há

1+
1+2+3+4+5+
1+2+3+4+5+6+7+8+9+
  4+5+6+7+8+9+10+9+8+7+
  8+9+10+9+8+7+6+5+4+3+
   8+7+6+5+4+3+2+1+
   4+3+2+1 =
=1+15+45+73+69+36+10=249 cubinhos do tipo A0. O absurdo é o mesmo.
=
Instruções para entrar na lista, sair da lista e usar a lista em
http://www.mat.puc-rio.br/~nicolau/olimp/obm-l.html
O administrador desta lista é [EMAIL PROTECTED]
=



[obm-l] Re: [obm-l] Re: [obm-l] Fatoração

2002-03-25 Por tôpico Ralph Teixeira

Problema:Fatorar x^10+x^5+1.

Resposta: Comece pensando em t=x^5 e notando que t^2+t+1 = (t^3-1)/(t-1) --
veja abaixo.
No segundo passo, fatorei o x^15-1, mas agora pensando em u=x^3 e
u^5-1 = (u-1)(u^4+u^3+u^2+1). Daí pra frente, é só rearrumar as coisas
cruzando os dedos para dar certo.

x^10+x^5+1 = (x^15-1)/(x^5-1) =
{(x^3-1)(x^12+x^9+x^6+x^3+1)}/{(x-1)(x^4+x^3+x^2+x+1)} =
= {(x^3-1)/(x-1)}{(x^12+x^9+x^6+x^3+1)/(x^4+x^3+x^2+x+1)} =
(x^2+x+1)(x^8-x^7+x^5-x^4+x^3-x+1)

--//--

Traduzindo em complexos: as raízes de x^10+x^5+1 são as raízes 15as da
unidade, tirando as raizes quintas, como a minha primeira igualdade acima
mostra. Isto é, se a é uma raiz primitiva de ordem 15 (digamos,
a=e^(2(Pi)i/15)), entao as raizes de x^10+x^5+1 sao

{a, a^2, a^4, a^5, a^7, a^8, a^10, a^11, a^13, a^14}

Mas a^5 e a^10 sao as duas raizes cubicas complexas da unidade! Juntas elas
sao raizes do polinomio

(x^3-1)/(x-1) = x^2+x+1 (=(x-a^5)(x-a^10))

E portanto x^2+x+1 divide x^10+x^5+1. O resto é fazer a conta da divisão e
torcer para dar tudo inteiro.

Abraço,
Ralph

=
Instruções para entrar na lista, sair da lista e usar a lista em
http://www.mat.puc-rio.br/~nicolau/olimp/obm-l.html
O administrador desta lista é [EMAIL PROTECTED]
=



RE: [obm-l] Re

2002-02-21 Por tôpico Ralph Teixeira

 
Oi, gente.

1)Seja f:R==R,não identicamente nula,tal que

f(x)*f(y)=(1/2)[f(x+y)+f(x-y)] e f(1)=0,para todos os números reais x e
y.

a)Mostre que f(0)=1,f(2)=-1,f(3)=0 e f(4)=1.
b)Mostre que f(x+4)=f(x),para todo x real.
c)Existe de fato tal função.

Bom, (a) saiu? Ok... Para (b), experimente fazer y=1 e prove que
f(x+1)=-f(x-1), isto eh, f troca de sinal de 2 em 2. Assim,
f(x+4)=-f(x+2)=f(x).

Existe tal funcao? Bom, funcao periodica assim, 1,0,-1,0,1,0,-1,0,... me
lembra senos e cossenos. Com um pouco de cuidado a gente tenta algo com
periodo 4, por exemplo:

f(x)=cos(Pi*x/2)

e ve que dah certo (isto eh, cos(Pi*x/2) satisfaz as condicoes do
enunciado... :PPronto, tal funcao existe (mas nao dissemos nada a
respeito dela ser unica!).

Abraco,
   Ralph
=
Instruções para entrar na lista, sair da lista e usar a lista em
http://www.mat.puc-rio.br/~nicolau/olimp/obm-l.html
O administrador desta lista é [EMAIL PROTECTED]
=



RE: [obm-l] divergente

2002-01-26 Por tôpico Ralph Teixeira

Oi, Paulo e galera.
 
Eu gosto de pensar de um jeito que fisicamente nao funciona bem... mas me
dah a ideia correta. Primeiro voce tem que entender bem o que eh o FLUXO de
um campo vetorial.

FLUXO: digamos que E eh um campo vetorial (pode ser no plano ou no espaco,
no que se segue, penso no espaco). Pense em E como um campo de velocidades
de agua (ou seja, a agua flui e, no ponto (x,y,z) a velocidade da agua eh o
vetor E(x,y,z)). Suponha que E eh um campo estacionario, isto eh, E pode
mudar de ponto para ponto, mas E NAO MUDA com o tempo, ok? Assim eu nao
preciso botar t em lugar algum...

Bom, pegue um ponto P(x0,y0,z0), coloque um cubinho imaginario (com
superficie feita de Perfex(TM)) em torno dele; voce sempre pode medir o
volume de agua que sai atraves do Perfex(TM) por segundo -- isto eh o FLUXO
de E atraves da superficie do cubinho (positivo se sai mais agua do que
entra, negativo caso contrario) --  a unidade disso eh algo como fluxo =
tantos litros / segundo, que saem do cubinho.

(Em geral, fluxo pode ser definido para qualquer superficie, nao soh
cubinhos)

--//--
DIVERGENTE: Se voce fizer o cubinho cada vez menor, o fluxo de agua que sai
do cubinho *tipicamente* vai para zero -- o cubinho eh pequeno demais, nao
hah Perfex(TM) suficiente atraves do qual possa sair agua. Alias,
*tipicamente*, se a eh o tamanho da aresta do cubinho, o fluxo atraves
dele eh parecido com ma^3 para algum numero m que depende de como eh o campo
vetorial E em volta do ponto P. Em outras palavras, tipicamente tem-se:

lim (a-0) FLUXO/a^3 = m(x0,y0,z0)

Esse m eh o chamado DIVERGENTE do campo E no ponto (x0,y0,z0). Em geral,
quando esse limite existe, voce nao PRECISAVA usar cubos; voce pode pegar
outros solidozinhos (S) em volta de P e fazer:

lim (V-0) (FLUXO atraves da superficie de S) / Volume de S

e este limite tambem dah o tal DIVERGENTE de E em P.

[[Alias, isso tudo nada mais eh do que uma DENSIDADE DE FLUXO... Se voce
quisesse densidade de massa, voce faria tudo do mesmo jeito, exceto que
lim (V-0) (MASSA dentro de S)/(Volume de S) = DENSIDADE DE MASSA
Troque massa por fluxo e veja a analogia...]]

SIGNIFICADO DO DIVERGENTE: Essa eh a definicao geometrica do divergente. O
que significa? Bom, no caso de agua, se div E (x0,y0,z0)  0 entao tem
volumes de agua saindo do ponto P(x0,y0,z0). Se tem agua saindo do ponto
P, eh como se P fosse uma fonte de agua, e o tamanho do divergente dah uma
ideia de quanta agua eh produzida em P por unidade de tempo. Se div E (P) 
0, P eh um ralo -- a agua entra no cubinho em volta de P e some ali
dentro!

Neste momento eh que voce percebe que a minha maneira de pensar nao eh muito
fisica... afinal, do jeito que eu falei, tem um anjo criando agua em alguns
lugares e tirando agua de outros -- como eh que a agua aparece no ponto
P?!? Mas eh assim que eu penso no divergente de E, como uma medida de agua
(cuja velocidade eh E) criada/destruida por anjos.

Sem anjos, no caso de um fluido incompressivel (a agua praticamente satisfaz
essa condicao em varias situacoes), o seu campo de velocidades E tem de ter
divergente NULO -- volumes de agua nao podem aparecer ou desparecer.

Compare a situacao da agua com um outro gas qualquer: se E for o campo de
velocidades de um gas (que eh um fluido compressivel), div E em P  0 quer
dizer que hah volumes de gas saindo de P. Mas voce nao precisa de um anjo
para tanto, basta que o gas esteja se EXPANDINDO para que voce detecte
volumes de gas sendo criados! Lembre-se, MASSA nao pode ser criada do
nada, mas VOLUME pode, desde que voce mude a densidade do fluido em questao.
Neste caso, div E (no ponto P) dah a ideia de quanto o gas estah se
expandindo no ponto P; divE0 quer dizer expansao (quanto maior o numero,
mais forte a expansao), divE=0 quer dizer que o *volume* de gas se mantem
(aa medida que se move, nos pontos onde divE=0), divE0 indica contracao.

Mais exatamente, se divE= 2 m^3/(segundo.m^3) = 2 /segundo (no sistema MKS)
em todos os pontos de uma certa regiao R do espaco, entao uma porcao de gas
que se mexe ali dentro vai dobrar de volume em 1 segundo (possivelmente
ocupando um OUTRO lugar ali dentro, mas o volume ocupado serah o dobro). Se
o div nao eh constante na regiao R, voce tem de fazer uma especie de media
para saber o volume de fluido que estah sendo criado (ou destruido): faca a
integral de div E na regiao R... Dai vem o teorema da divergencia:

INT (tripla, dentro de R) div E dV =
= Fluxo de E (atraves da fronteira/superficie de R)

(Isto eh, Integral da densidade de fluxo = fluxo total; compare com Integral
da densidade de massa = massa total)

Para eletricidade, note que um campo eletrico tambem pode sim aparecer ou
desaparecer -- a presenca de CARGA eletrica constitui uma fonte ou um ralo
de campo eletrico (apesar de agora E nao ser um campo de velocidades... mas
pra mim a analogia ainda ajuda). Quanto maior a densidade de carga num
lugar, maior a quantidade de campo eletrico que sai daquele lugar -- a
relacao exata eh a 

Re: [obm-l] russos

2002-01-24 Por tôpico Ralph Teixeira

1)Prove que em qualquer sequencia de 39 numeros naturais consecutivos
existe ao menos  um numero cuja a soma dos algarismos e divisivel por
11.
Hmmm...que tal assim:

Caso (1) Se nao houver troca de centena entre esses 39 numeros

Neste caso, a observacao chave eh a da Iolanda: a soma dos algarismos de n
na aumenta de 1 a cada vez que o numero aumenta de 1; exceto quando hah
troca de dezena, quando entao a soma diminui em 8 (-9 nas unidades, +1 nas
dezenas). Assim, a sequencia dos restos das somas dos algarismos de n na
divisao por 11 serah um subconjunto da seguinte sequencia ciclica:

0 1 2 3 4 5 6 7 8 9
1 2 3 4 5 6 7 8 9 10
2 3 4 5 6 7 8 9 10 0
3 4 5 6 7 8 9 10 0 1
4 5 6 7 8 9 10 0 1 2
...
10 0 1 2 3 4 5 6 7 8

(a partir daqui a sequencia repete -- note que eu troco de linha ao trocar
dezenas, assim cada coluna corresponde a uma possibilidade de digito final
para n). Note que existem 28 restos entre os dois primeiros zeros; entre os
outros pares consecutivos de zeros hah apenas 8 restos nao nulos. Como a
nossa sequencia tem 39 desta sequencia de restos, pelo menos um 0 aparece.

Caso (2) Hah uma troca de centena na sequencia (possivelmente uma troca de
ordem maior ao mesmo tempo, nao me interessa)
Este eh o ultimo caso -- 39 numeros consecutivos nao podem trocar as
centenas DUAS vezes...
 Neste caso, os 39 numeros sao divididos em duas subsequencias, uma antes da
grande troca e outra depois. A sequencia de restos que vem antes da GRANDE
TROCA serah uma subsequencia da tabela ciclica acima, soh que agora o ultimo
elemento TEM DE ESTAR NA ULTIMA COLUNA (que corresponde aos numeros que
terminam por 9) -- chamemos isso
de subsequencia do tipo (i). Analogamente, a sequencia de restos que comeca
com o resto do numero 100k, TEM DE COMECAR NA PRIMEIRA COLUNA -- tipo (ii).

Nao eh dificil ver que a maior sequencia do tipo (i) sem zeros eh a que
comeca no primeiro 1 e vai ateh o 10 da segunda linha (com 19 elementos); a
maior possivel do tipo (ii) sem zeros COMECA no 1 da segunda linha e termina
no 10 da terceira, com mais 19 elementos. Como temos 39 elementos  19+19,
concluimos que ao menos um numero da lista tem soma dos algarismos divisivel
por 11.

---///---

Alias, com essa tabela em mãos, não é difícil encontrar uma sequencia com 38
elementos sem somas de algarismos divisiveis por 11. Teremos de usar algo da
forma 100k-19, 100k-18,,100k-1, 100k, 100k+1,...,100k+18 onde a soma dos
algarismos de 100k-1 deixa resto 10 na divisao por 11 e a soma dos
algarismos de 100k deixa resto 1 na divisao por 11.

Se 100k termina por m zeros, entao sao m noves que somem de 100k-1 para
100k, entao a soma dos algarismos diminui em 9m-1 (como a Iolanda jah tinha
observado). Para conseguir que o resto pule de 10 para 1, precisamos de que
9m-1=9 modulo 11, isto eh, m seja da forma 11a+6 -- temos de terminar com
11a+6 zeros em 100k. Assim, a sequencia de 38 elementos sem soma dos
algarismos divisivel por 11 comecando pelo menor numero eh:

81 82 83 ... 99 100 101 ... 118

Legal?

Abraco,
 Ralph

=
Instruções para entrar na lista, sair da lista e usar a lista em
http://www.mat.puc-rio.br/~nicolau/olimp/obm-l.html
O administrador desta lista é [EMAIL PROTECTED]
=



Fw: [obm-l] Duvida em exponencial

2002-01-22 Por tôpico Ralph Teixeira

 Oi, Vicente.

 Que eu saiba, nao existe maneira de resolver essa equacao para n
usando as funcoes aas quais a gente tah acostumado...

 Mas existe uma tal funcao W de Lambert que se define assim:

 LambertW(y)=x quando x.e^x=y

 (note que e^LambertW(y) = y/LambertW(y), por definicao)

 Essa funcao estah bem definida para y=0; para -1/ey0 voce tem
dois possiveis valores de x para cada valor de y, um entre -1 e 0 e o
outro menor que -1. Para y=-1/e, x=-1 eh a unica solucao. Enfim, nao hah
solucao para y-1/e.

 De qualquer forma, usando esta criatura, dah para resolver x^x=y da
seguinte forma (pelo menos quando y0):

 x^x=y
 x lnx = lny
 lnx e^(lnx) = lny
 lnx = LambertW(lny)
 x = e^LambertW(lny)
 x = lny/LambertW(lny)

onde na ultima passagem eu usei a propriedade que eu citei lah perto da
definicao.

 Bom, agora nao vejo saida senao ir numericamente. No seu caso:

 x^x=2^100
 x = ln(2^100)/LambertW(2^100)
 x = 100 ln2 / LambertW(100 ln2)
 Agora ln2~0.6931471806, e LambertW(693147.1806)~11.046852, entao:
 x ~ 693147.1806/11.046852 ~ 62746.12645

 Eu sei que essa resposta nao eh muito satisfatoria (essa funcao
LambertW eh muito esquisita e eu nem conheco nenhuma propriedade legal
dela exceto as citadas acima), mas nao creio que haja nada melhor. Pelo
menos, os numeros envolvidos no calculo numerico sao MUITO menores do
que trabalhar direto com 2^100 :) :) :)

 Abraco,
  Ralph

-Original Message-
From: Vicente
To: [EMAIL PROTECTED]
Sent: 1/22/02 12:23 AM
Subject: [obm-l] Duvida em exponencial

Bem, eu tava resolvendo um problema com logaritmos e cheguei no seguinte
resultado:

n^n=2^10^6 (ou 2¹°°)

Existe algum cálculo utilizado para igualar a base ao expoente???


Obrigado.

Vicente.


=
Instruções para entrar na lista, sair da lista e usar a lista em
http://www.mat.puc-rio.br/~nicolau/olimp/obm-l.html
O administrador desta lista é [EMAIL PROTECTED]
=



Re: Re:Exercicios (Solução do exerc´cio 2)

2002-01-16 Por tôpico Ralph Teixeira




  2)Um quadrilátero ABCD está inscrito em um 
circulo de raio 5 , tal que :  AB=4 BC=6 CD=X AD=8 Qual 
o valor de X ?
 Aqui vai outra solução, que 
é equivalente à solução do Ponce.Eu a coloco aqui para dar idéias na hora 
de fazer outros problemas com quadriláteros inscritos -- veja se vale a pena 
alterar a ordem dos vértices!
Quero dizer com isso: 
considere o quadrilateroAXYD inscrito no mesmo círculo onde A eD são 
os mesmos de antes, mas AX=6, XY=CD e YD=4 (AD=8 ainda) -- note que isto é 
possível, basta tomar os mesmos arcos de antes e transportá-los para os lugares 
corretos; como antes os arcos somavam 360, ainda somam 360.

 Agora, como AX^2+AD^2=100=10^2 
(diâmetro^2), o trianguloDAX é retângulo em A; entao XYD tambem é 
retângulo, e então:

 x^2+4^2=100 = 
x=sqrt(84)=2sqrt(21)

 Como na solução do 
Ponce.

 Abraço,
 
Ralph


Pontos Gordos

2002-01-15 Por tôpico Ralph Teixeira

 Oi, galera.

Tem algo estranho aqui... Considere dois feixes de retas passando pelos
pontos A(-1,0) e B(1,0), feixes estes simetricos com relacao ao eixo Oy e
portanto se intersectando lah em varios pontos (digamos, os pontos da forma
(0,n) com n natural). Junte estes dois feixes com o eixo Oy. Entao:

i) Duas retas do feixe A se encontram em A, que eh GORDO.
ii) Duas retas, uma de cada feixe, se encontram em um ponto do eixo Oy, que
eh GORDO (pois o eixo Oy tambem passa lah);
iii) Uma reta de um feixe e o eixo Oy se encontram em pontos GORDOS (pois a
outra reta do outro feixe tambem passa lah).

Assim, o numero minimo de pontos MAGROS eh zero, como mostra o meu
exemplo... Eu entendi a questao direito?

Abraco,
 Ralph


Segue um problema de uma lista de seleção pra imo-ibero do ano passado
:
Considere um número finito de retas coplanares. Um ponto magro de 
intersecção é um ponto onde concorrem exatamente 2 retas. Supondo que 
existem pelo menos 2 pontos de intersecção, determine o número mínimo
de 
pontos magros de intersecção.
Não sei o nível de dificuldade. aguardo respostas..
Abraços,
 Villard




_
Converse com amigos on-line, experimente o MSN Messenger: 
http://messenger.msn.com.br



Determinante (Era: Por favor, me tirem dessa lista.)

2002-01-11 Por tôpico Ralph Teixeira

Eu tenho uma idéia, Nicolau: se alguém mandar uma mensagem para a lista
pedindo para sair, a gente os pune! Eu sugiro pena máxima: a gente os
EXPULSA da lista! Isso mesmo!

:) :) :)

---///---

Para pagar a piada off-topic, devo ser obrigado a incluir um problema,
certo? Aqui vai um problema de Álgebra Linear bacaninha do curso que eu
acabei de dar; a solução mais curta ganha um E-doce

Sejam A uma matriz m x n e B uma matriz n x m. Mostre que
det[0 A; -B I] = det(AB)

(Notação: eu uso [a b c; d e f; g h i] para representar uma matriz 3x3 cujas
linhas são [a b c], [d e f], [g h i]; a identidade 3x3, por exemplo, é [1 0
0; 0 1 0; 0 0 1]. Aqui, a matrizona (m+n) x (m+n)  do lado esquerdo é
montada por blocos; 0 é a matriz nula m x m e I é a matriz identidade n x
n.)

Abraço,
Ralph




Re: questões do livro Curso de Análise ,vol1 , AJUDA.

2002-01-11 Por tôpico Ralph Teixeira



Pequena correção: x^2+y^2-z=0 é um parabolóide de 
revolução; para ser cone, precisava ser x^2+y^2=z^2.

Abraço, Ralph.

  - Original Message - 
  From: 
  David 
  Daniel Turchick 
  To: [EMAIL PROTECTED] 
  Sent: Thursday, January 10, 2002 1:28 
  PM
  Subject: Re: questões do livro Curso de 
  Análise ,vol1 , AJUDA.
  
  Bom, o primeiro eu fiz...
  Queremos a imagem inversa do ponto 0, pela 
  função f (i.e., o conjunto de todos os pontos do domínio de f que são levados 
  no 0).
  Então (x,y) Î f^(-1) (0) sse 
  f(x,y)=0, i.e, 3x-y=0, i.e., y=3x. Logo f^(-1) (0) = {(x,y) Î RxR / y=3x} = 
  {(x,3x) Î RxR} = [(1,3)] (o conjunto gerado pelo vetor (1,3)). Logo, o 
  conjunto pedido é a reta passando pela origem com inclinação 
  3.
  Analogamente: 
  
  g^(-1) (0) = circunferência centrada em (1,1) de 
  raio 3.
  w^(-1) (0) = 
  plano xy.
  u^(-1) (0) = 
  cone... (acho que não sei descrever com palavras, mas fazendo a figura fica 
  muito fácil de ver)
  
-Mensagem original-De: 
haroldo [EMAIL PROTECTED]Para: 
[EMAIL PROTECTED] [EMAIL PROTECTED]Data: 
Quinta-feira, 10 de Janeiro de 2002 13:28Assunto: En: 
questões do livro Curso de Análise ,vol1 , AJUDA.

-Mensagem original-De: 
haroldo [EMAIL PROTECTED]Para: 
[EMAIL PROTECTED] [EMAIL PROTECTED]Data: 
Terça-feira, 8 de Janeiro de 2002 23:22Assunto: questões do 
livro Curso de Análise ,vol1 , AJUDA.
Saudações a todos os amigos da lista.
Gostaria de ajuda em 2 exercícios do livro do 
Elon.

1-Sejam f,g : RxR - R e w,u : RxRxR R as 
funções definidas por f(x,y)= 3x-y, g(x,y)=(x-1)^2 + (y-1)^2 -9 , w(x,y,z)= 
3z,
u(x,y,z)= x^2 + y^2 - z . interpretando (x,y) como as 
cooredenadas de um ponto do plano RxR e (x,y,z) como coordenadas de um 
ponto do espaço, descreva geometricamente os conjuntos f^(-1) (0) , g^(-1) 
(0) , w^(-1) (0) , z^(-1) (0).

2-Um número real chama-se transcedente quando não é 
algébrico . prove que o conjunto dos números transcedentes é não -enumerável 
e denso em R.


Re: Idade

2002-01-07 Por tôpico Ralph Teixeira



Oi, Pedro.

Argh Eu tenho 
uma relacao de amor-ódio com esses problemas... :)

Bom, vamos lá organizar tudo. Seja x a DIFERENÇA 
entre as idades (queé a mesma em todas as épocas). Organize tudo por épocas:

(1) Eduardo tem dobro de anos
(2) que João tinha quando Eduardo tinha a metade da 
idade
(3) que João terá quando João tenha (hmmm... 
tiver?) o triplo da idade
(4) que Eduardo tinha quando Eduardo tinha o dobro 
da idade do João naquela época.

(1) Presente ("tem")
(2) Passado I ("tinha")
(3) Futuro ("terá")
(4) Passado II (outro "tinha"; "naquela 
época")

Agora escrevemos as equações ao 
contrário.

(4) Bom, Eduardo tinha o dobro da idade do Joao 
naquela epoca. Como a diferenca eh x, as idades eram
Joao:x e Eduardo:2x naquela epoca. Note que Eduardo 
eh mais velho.
(3)Joao terá o triplo da idade que Eduardo 
tinha em (4). Então aí João:6x.
(2) Eduardo tinha a metade da idade de João em (3). 
Então Eduardo:3x e consequentemente João:2x.
(1) Eduardo tem o dobro da idade de João em (2). 
Então Eduardo:4x e consequentemente Joao:3x.

Como 4x+3x=70, x=10. João tem 30 anos e Eduardo tem 
40 anos.

Verificando Eduardo tem (40 ANOS)
o dobro do que João tinha (20 anos) quando Eduardo 
tinha (30 anos)
a metade da idade que João terá (60 anos) quando 
João tiver
o triplo da idade que Eduardo tinha (20 anos) 
quando Eduardo tinha o dobro da idade de João (na 
época, 10 anos).

Abraço,
  
Ralph


RE: ajuda em análise

2002-01-06 Por tôpico Ralph Teixeira

 
 Esse eh um exercicio bem bonito que eu vi pela primeira vez no livro de
Analise do Elon A ideia eh simples, mas mais facil de explicar com uma
figurinha Bom, eu explico a solucao e voce faz a figurinha, que tem os
conjuntos A e B, setas de A para B que representam f e setas de B para A que
representam G. :)

 Dado um elemento a0 de A, veja se hah um elemento b0 de B tal que 
g(b0)=a; entao veja se hah um elemento a1 de A tal que f(a1)=b0; entao veja
se hah um elemento b1 de B tal que g(b1)=a1; e assim por diante...
Basicamente, voce cria uma sequencia de elementos que estao alternadamente
em A e B usando alternadamente as inversas de f e g enquanto isso for
possivel (na figurinha, dah um zigue-zague no sentido contrario ao das
setas). Note que, como f e g sao injetivas, a escolha desta cadeia a partir
de um certo elemento eh unica. Tres coisas podem acontecer:


i) Essa cadeia pode terminar num elemento an de maneira que nao existe
bn em B tal que g(bn)=an (talvez ateh logo no primeiro elemento a0). Neste
caso, defina h em toda a cadeia assim: h(a_i)=f(a_i). Note que eu cobri
todos os b's desta cadeia, e defini h para todos os a's dela...

ii) Essa cadeia pode terminar num elemento bn de maneira que nao existe
an em A tal que f(an)=bn. Neste caso, defina h em toda a cadeia assim:
h(a_i)=g^(-1)(a_i)=b_i. Note que eu cobri todos os b's desta cadeia de novo,
e todos os a's foram usados pois todos eles tem inversos pela g.

iii) Essa cadeia pode nao terminar nunca (sendo ciclica ou nao). Neste
caso, faca como quiser... Por exemplo, defina h(a_i)=f(a_i) como no caso
(i). Eu tambem cobri todos os a's e b's aqui.

Pronto. Essa funcao h eh agora uma bijecao de A em B. De fato, todo
elemento b de B estah numa destas cadeias (na cadeia iniciada por g(b), por
exemplo), e portanto ela eh sobrejetiva. A injecao segue da unicidade da
cadeia: se voce comecar de b em B, a cadeia iniciada por g(b) eh unica, e
deve terminar num elemento de A ou de B, e portanto obedece apenas a um dos
casos anteriores.

Minha explicacao eh mais complicada do que a ideia Funcionou?

Abraco,
 Ralph



-Original Message-
From: haroldo
To: [EMAIL PROTECTED]
Sent: 1/6/02 5:59 PM
Subject: En: ajuda em análise

 
-Mensagem original-
De: haroldo  [EMAIL PROTECTED] mailto:[EMAIL PROTECTED] 
Para: [EMAIL PROTECTED] mailto:[EMAIL PROTECTED]  
[EMAIL PROTECTED] mailto:[EMAIL PROTECTED] 
Data: Domingo, 6 de Janeiro de 2002 18:34
Assunto: ajuda em análise


Saudações a todos .
Alguém poderia me ajudar na seguinte questão :
Dados os conjuntos A e B , suponha que existam funções injetivas f: A -
B e g: B-A . Prove que existe uma bijeÇão h:A-B.



RE: Sistema Poli 1942

2002-01-06 Por tôpico Ralph Teixeira

1) Resolver (imagino que para x e y, certo?):

tg2x + tg2y = a
tgx + tgy = b

--//--

Vejamos Uso A=tgx e B=tgy, entao a primeira eh:

2A/(1-A^2)+2B/(1-B^2) = a
2A(1-B^2)+2B(1-A^2)=a(1-A^2)(1-B^2)
2A+2B-2AB(A+B)=a(1-A^2-B^2+A^2B^2)

Use agora S=A+B=b e P=AB. Note que A^2+B^2=S^2-2P. Entao:

2b-2Pb=a(1-b^2+2P+P^2)
aP^2+2P(a+b)+a-2b-ab^2=0
P^2+2P(1+b/a)+1-2b/a-b^2=0

(escreverei daqui por diante r=b/a)

P^2+2P(1+r)+1-2r-b^2=0
P=-(1+r)+-sqrt(r^2+b^2+4r)

Enfim, como A e B sao as duas raizes de t^2-bt+P=0, tem-se
t=b/2+-sqrt(b^2/4-P), isto eh:

{tgx,tgy}=b/2+-sqrt(b^2/4+1+r-sqrt(r^2+b^2+4r))
ou
{tgx,tgy}=b/2+-sqrt(b^2/4+1+r+sqrt(r^2+b^2+4r))

Eh... O meu tambem ficou feissimo... E como aquela raiz dupla nao parece se
desdobrar em uma soma de raizes, nao vai melhorar muito nao. E como a
resposta eh MUITO feia, nao creio que haja solucao que melhore muito isso
nao... :(

Abraco,
   Ralph




RE: Matriz

2002-01-06 Por tôpico Ralph Teixeira

   Oi, Emanuel.

   De fato, ha algo errado ai... Se for de fato

Aij = -1 se i=j
Aij = i+j se i=j

Entao A11=-1=2 ao mesmo tempo, absurdo Provavelmente queria-se que
apenas um dos dois casos contivesse a igualdade. Se soh o de cima tivesse
igualdade, seria A = [-1 -1; 3 -1]; caso contrario (soh o debaixo com
igualdade), seria A = [2 -1; 3 4]. Os dois ao mesmo tempo nao dah.

Abraco,
 Ralph




Re: beal

2001-12-17 Por tôpico Ralph Teixeira

Eu achei isso aqui procurando via Google (procure Beal Conjecture)
Há outros links, esse é o primeiro:

http://www.math.unt.edu/~mauldin/beal.html

Quem nao quiser ir lá, basicamente essa página diz

THE BEAL CONJECTURE AND PRIZE

BEAL'S CONJECTURE: If A^x +B^y = C^z , where A, B, C, x, y and z are
positive integers and x, y and z are all greater than 2, then A, B and C
must have a common prime factor.

THE BEAL PRIZE. The conjecture and prize was announced in the December 1997
issue of the Notices of the American Mathematical Society. Since that time
Andy Beal has increased the amount of the prize for his conjecture. The
prize is now this: $100,000 for either a proof or a counterexample of his
conjecture. The prize money is being held by the American Mathematical
Society until it is awarded. In the meantime the interest is being used to
fund some AMS activities and the annual Erdos Memorial Lecture.

CONDITIONS FOR WINNING THE PRIZE. The prize will be awarded by the prize
committee appointed by the American Mathematical Society. The present
committee members are Charles Fefferman, Ron Graham, and Dan Mauldin. The
requirements for the award are that in the judgment of the committee, the
solution has been recognized by the mathematics community. This includes
that either a proof has been given and the result has appeared in a
reputable refereed journal or a counterexample has been given and verified.

PRELIMINARY RESULTS. If you have believe you have solved the problem, please
submit the solution to a reputable refereed journal. If you have questions,
they can be mailed to:

The Beal Conjecture and Prize
c/o Professor R. Daniel Mauldin
Department of Mathematics
Box 311430
University of North Texas
Denton, Texas 76203

Questions and queries can also be FAXED to 940-565-4805 or sent by e-mail to
[EMAIL PROTECTED]


O Morgado agora pode dormir em paz. :) :)

Abraço,
Ralph

- Original Message -
From: Augusto César Morgado [EMAIL PROTECTED]
To: [EMAIL PROTECTED]
Sent: Monday, December 17, 2001 5:16 PM
Subject: Re: beal


Pelo amor de Deus, não consigo dormir de curiosidade. Sobre qual assunto
é essa conjectura de Beal? Internautas, ninguém descobriu nada sobre
esse cara?





Re: Quantidade de oleo

2001-11-27 Por tôpico Ralph Teixeira



 Engracado... Alguem me fez 
exatamente esta pergunta, por E-mail, alguns meses atras (se eu me lembro bem, 
por motivos praticos, havia ateh os valores numericos envolvidos); a unica 
diferenca eh que no caso que eu tentei analisar, eram calotas esfericas de raio 
R nas bordas, nao necessariamente hemisferios. A recomendacao no caso era 
simplesmente colocar um certo volume conhecido no tanque, ver a altura, e fazer 
a "marquinha" daquele volume na vareta. Repita para varios volumes e faca a sua 
escala sem fazer conta alguma... :)

 Ou, melhor ainda, faca um modelo 
em escala para realizar o processo acima e depois marque a varetona em escala. 
:) A gente que gosta de matematica muitas vezes esquece que uma resposta 
numerica (ou um grafico) pode frequentemente ser tao boa quanto ou melhor que 
uma "formula". Mas no caso do problema aqui,a gente quer uma formula, 
certo?

 Infelizmente, eu nao estou com 
aquele E-mail aqui, saiu de outra conta Se ninguem se dispuser a responder, 
eu devo conseguir uma copia dele 5a feira para mandar para a lista. 
Eu*acho* que era uma conta bem feia, especialmente as integrais nas 
calotas esfericas, que no caso geral (raio R) nem se resolvia no braco... Mas 
nesse caso (raio da esfera = raio do cilindro), acho que saia algo mais 
bonitinho. Sugestao para a galera: faca a conta toda usando z=h-r; isto eh, ao 
inves de usar h, faca as contas com z onde z=0 indica metade do tanque cheio -- 
as contas intermediarias ficam bem mais simples. No final apenas, troque z por 
h.

 Abraco,
  
 Ralph

  - Original Message - 
  From: 
  Davidson 
  Estanislau 
  To: obm 
  Sent: Tuesday, November 27, 2001 9:37 
  AM
  Subject: Quantidade de oleo
  
  
  
   Como faço para determinar a quantidade de óleo que 
  há em um caminhão que abastece os postos de gasolinas, dispondo apenas de uma 
  vareta? Essa vareta será usada, para determinar a altura do óleo existente no 
  reservatório do caminhão
   Sabendo que o reservatório é formado pela união de 
  um cilindro com duas semi-esferas nas extremidades.
   Vejam um 
  esboco do reservatorio:
  
   
  
  
   Dados: 
   Altura medida pela vareta: h;
   Raio das semi-esferas: r;
   Distância entre as extremidades das semi-esferas: 
  H.
  
   Ogrigado pela ajuda.
   Davidson Estanislau


RE: Ternas (x,y,z)

2001-11-22 Por tôpico Ralph Teixeira

 

Oi, Yuri.

Note que as ternas do tipo (a,a,b), (a,b,a) ou (b,a,a) para quaisquer a e b
reais satisfazem as equacoes. (Isto eh, x=y OU z=x OU y=z satisfazem a
equacao dada).

--//--

Agora, o que eu vou fazer aqui a seguir serve se n eh par OU voce assumir
que x,y,z sao positivos.

Se x=y, x=z ou y=z, coom jah dissemos, a equacao vale. Caso contrario, posso
supor que yz sem perda de generalidade. Considere:

f(x)= y*x^n + z*y^n + x*z^n - (x*y^n + y*z^n + z*x^n) =
= x^n (y-z) -x (y^n-z^n) + (z*y^n-y*z^n)

O truque eh mostrar que as unicas raizes reais deste polinomio em x sao de
fato x=y e x=z. 

Se voce quer soh solucoes positivas, um jeito de mostrar isso eh dividir
f(x) por (x-y)(x-z)(y-z) e notar que o polinomio (HORRIVEL) que fica tem
todos os coeficientes positivos Assim, nao ha como ter raizes positivas
dele. Deve ter um jeito mais facil de mostrar que f(x) eh (crescente e)
positivo para xy sem calculo, mas eu nao achei ainda.

Usando calculo, eu termino a questao um pouco mais facil. Repito, a solucao
a seguir serve se n eh par OU se limitamo-nos a x,y,z0.

f'(x) = n*x^(n-1)(y-z)-(y^n-z^n)
f''(x)=n(n-1)x^(n-2)(y-z)

Se n=1, eh claro que f(x)=0 e qualquer terna (x,y,z) satisfaz a equacao
dada.

Senao, se n for par OU se a gente olhar apenas no intervalo x \in (0,+INF)
dah para ver entao que f''0 e portanto a funcao f eh convexa! Assim, f tem
no maximo duas raizes (voce podia tambem usar o Teorema de Rolle direto de
notar que f'(x)=0 soh tem uma raiz, dah no mesmo). Como x=y e x=z sao duas
raizes de f(x), concluimos que sao as unicas. Assim, as unicas raizes reais
de f sao as dadas acima.

---///---

Este raciocinio todo fura se n for impar E a gente tiver que achar raizes
reais onde uma ou mais variaveis possam ser negativas! Neste caso, o
problema eh nao trivial; creio eu que o enunciado original deveria ter algo
mais a esse respeito. Soh para ter uma ideia de como a coisa complica se
voce permitir n impar E respostas negativas:

i) Para n=3, eh facil fatorar f(x)=(x-y)(y-z)(z-x)(x+y+z); assim, alem das
raizes x=y, x=z ou y=z, tambem temos qualquer terna que satisfaca x+y+z=0 (e
aqui pelo menos um dentre x, y ou z tem de ser nao positivo).

ii) Para n=5, fatore f em
(x-y)(y-z)(z-x)((x+y+z)(x^2+y^2+z^2)+xyz)
Os tres primeiros termos sao os esperados, e o terceiro nao tem raizes se x,
y e z forem nao negativos. Mas se eles podem ser negativos bom, a
equacao

(x+y+z)(x^2+y^2+z^2)+xyz=0

define uma superficie muito doida no R^3, que eu ateh plotei aqui no meu
Maple da vida Assim, HA solucoes sim, e nao dah para escreve-las dum
jeito bonitinho (que eu saiba).

iii) Para n=7,9,11, etc... com solucoes negativas a coisa piora, entao eu
nem fiz. :)

Ajudou?

Abraco,
 Ralph

-Original Message-
From: [EMAIL PROTECTED]
To: [EMAIL PROTECTED]
Sent: 11/22/01 12:02 AM
Subject: Ternas (x,y,z)

 Alguém pode resolver ou dar dicas para a seguinte questão???

 Dado um inteiro positivo n, achar todas as ternas (x,y,z) de números
reais
tais que
 y*x^n + z*y^n + x*z^n = x*y^n + y*z^n + z*x^n

[]'s, Yuri
ICQ: 64992515


--
Use o melhor sistema de busca da Internet
Radar UOL - http://www.radaruol.com.br



-Original Message-
From: Paulo Santa Rita
To: [EMAIL PROTECTED]
Sent: 11/22/01 2:29 PM
Subject: Re: Ternas (x,y,z)

Ola Yuri e demais
colegas desta lista,

Se N=1, evidentemente qualquer terna (x,y,z) de numeros reais satisfaz a

equacao. Se N  1, a equacao pode ser colocada como um produto escalar,
da 
seguinte forma :

(y,z,x).(x^N,y^N,z^N)=(x,y,z).(y^N,z^N,x^N)

Agora, note o seguinte :

1) Como todos os modulos ( norma euclidiana ) sao iguais, segue portanto
que 
o produto sera igual se os angulos entre os vetores que estao sendo 
multiplicados forem iguais ou somarem 360 graus.

2) A funcao que e aplicada de um membro para o outro e :
T(y,z,x)=(x,y,z). 
Esta funcao, alem de ser biunivoca e portanto admitir uma inversa 
corresponde a uma operacao geometrica ( com os eixos coordenados ) bem 
definida.

Salvo por um melhor juizo dos demais colegas e Professores, EU ACHO que
com 
estas dicas a questao fica resolvida.

Um abraco pra voce
Paulo Santa Rita
5,1526,221101






From: [EMAIL PROTECTED]
Reply-To: [EMAIL PROTECTED]
To: [EMAIL PROTECTED]
Subject: Ternas (x,y,z)
Date: Thu, 22 Nov 2001 01:02:21 -0200

  Alguém pode resolver ou dar dicas para a seguinte questão???

  Dado um inteiro positivo n, achar todas as ternas (x,y,z) de números 
reais
tais que
  y*x^n + z*y^n + x*z^n = x*y^n + y*z^n + z*x^n

[]'s, Yuri
ICQ: 64992515


--
Use o melhor sistema de busca da Internet
Radar UOL - http://www.radaruol.com.br





_
Chegou o novo MSN Explorer. Instale já. É gratuito! 
http://explorer.msn.com.br



Re: IME (era: Re:dúvida)

2001-11-14 Por tôpico Ralph Teixeira

sqrt(5-sqrt(5-x))=x

Mknha solução é uma mistura de tudo o que você falou No braço, elevando
ao quadrado e tal:

5-sqrt(5-x)=x^2
5-x^2=sqrt(5-x)
25-10x^2+x^4=5-x
x^4-10x^2+x+20=0

Agora note que as raízes de sqrt(5-x)=x são raízes da equação original,
certo (eu não disse TODAS)? Isto dá uma dica de que o meu polinômio de
quarto grau deve ser divisível por x^2+x-5 De fato, aquela equação se
torna:

(x^2+x-5)(x^2-x-4)=0

E agora é fácil achar as 4 raízes

x1=(-1+sqrt(21))/2
x2=(-1-sqrt(21))/2
x3=(1+sqrt(17))/2
x4=(1-sqrt(17))/2

Mas o processo de elevar ao quadrado pode introduzir raízes estranhas!
Por exemplo, olhe a equação original e note que x=0, o que invalida x2 e
x4. De fato, para reverter os passos onde elevamos ao quadrado, temos de
verificar duas coisas:

i) 5-x^2=0
ii) x=0

Para x1, note que x1=5, então sqrt(5-x1)=x1; portanto
sqrt(5-sqrt(5-x1))=sqrt(5-x1)=x1 satisfaz a equação.
Para x3, note que x3^2=(18+2sqrt(17))/4=20/4=5, e, portanto, 5-x3^2=0;
assim, x3 não serve!

Assim, a única solução é x1=(sqrt(21)-1)/2.

Abraço,
Ralph




Re: Horas

2001-10-31 Por tôpico Ralph Teixeira


É, mas cuidado: entre 8 e 9 horas (exclusive) não há nenhuma posição com
o ponteiro das horas antes ...

- Original Message -
From: Eduardo Grasser [EMAIL PROTECTED]
To: [EMAIL PROTECTED]
Sent: Wednesday, October 31, 2001 12:01 PM
Subject: RE: Horas


Vejamos, com o ponteiro das horas antes, temos das 00:15 (mais uns
quebradinhos, mas não vem ao caso) às 23:10 dando 12 ângulos retos.
Com o ponteiro dos minutos antes, temos das 00:45 às 23:40 (horários
aproximados) dando 12 ângulos retos.

Total 24 retos.

eduardo

 --
 De: Frederico Gomes Elihimas[SMTP:[EMAIL PROTECTED]]
 Enviada em: Quarta-feira, 31 de Outubro de 2001 09:13
 Para: [EMAIL PROTECTED]
 Assunto: Horas

 Quantos angulos retos existem das 00:00 `as 24:00 formados pelos
 ponteiros
 dos minutos e das horas ?  nao eh permitido responder por fi'sica.









Re: Torneio das Cidades

2001-10-25 Por tôpico Ralph Teixeira


Eu gostei dessa questao e comecei a pensar um bocado nela Veja se
isso aqui funciona

i) O assistente escolhe duas cartas do mesmo naipe para serem
indicadoras principais (sempre hah duas cartas do emsmo naipe), digamos, A
(a maior das duas) e B (a menor). Ele calcula x=A-B (usando As=1, J=11, Q=12
e K=13).-- Se x=7 ele poe A na mesa e B no bolso
-- Se x=6 ele poe B na mesa e A no bolso.
De qualquer forma, a da mesa vem aa esquerda da fila na mesa.

ii) As outras 3 cartas na mesa indicarao um numero n de 1 a 6 (hah 6
permutacoes possiveis delas). Para tanto, basta criar uma ordem, digamos a
ordem dos naipes do bridge (Espadas  Copas  Ouros  Paus) com criterio de
desempate dado por AsKQJ10...2. Assim, se elas forem abc, entao:
abc=1 acb=2bac=3bca=4cab=5cba=6
O numero indicado serah n=x ou n=13-x (o que der para indicar, a primeira
opcao se x=6, a segunda se x=7).

Como eh que o adivinhante recupera a carta do bolso? Bom, ele faz o
seguinte:

i) A primeira carta indica o naipe pedido com certeza. Seu numero,
digamos, y, eh um excelente ponto de referencia, mas ele nao sabe ainda se
esta eh a menor ou a maior das duas cartas principais.
ii) Ele le o numero indicado pelas proximas 3 cartas, e descobre n.

Agora, considere as duas hipoteses:
-- Se y, a da mesa, fosse a menor das duas principais (y=B), entao o
metodo diz que n=x=6, certo? Assim, a carta do bolso seria A=x+B=y+n;
-- Caso contrario, y seria a maior das duas (y=A), e o metodo diz que
isso soh acontece se x=13-n. NEste caso, teriamos que a do bolso seria
B=A-x=y-(13-n)=y+n-13

Mas o truque eh que apenas uma dessas duas hipoteses dah um numero
valido! Assim, o adivinhante nao tem que fazer o raciocinio todo destas duas
linha; basta que ele faca o seguinte:

iii) Ele soma o numero da primeira carta da mesa (y) com o numero
indicado pelas outras 3 (n), fazendo isso modulo 13. Esse eh o numero da
carta do bolso!

Uau! Vou combinar isso com a minha esposa e fazer com os filhos dela
:) :) :)

Abraco,
Ralph
- Original Message -
From: Paulo Jose Rodrigues [EMAIL PROTECTED]
To: [EMAIL PROTECTED]
Cc: [EMAIL PROTECTED]
Sent: Wednesday, October 24, 2001 10:44 PM
Subject: Torneio das Cidades


Já que o assunto Torneio das Cidades é a bola da vez vão aqui alguns
esclarecimentos:

O Torneio das Cidades é uma competição organizada pela Rússia cujo
regulamento tenta permitir que cidades grandes e pequenas participem nas
mesmas condições (lembre que essa é uma competição entre cidades...)

As provas são corrigidas pelo comitê organizador de cada cidade e as
melhores enviadas para Moscou que emite os diplomas de premiação. O diploma
é da Academia de Ciências da Rússia, obviamente é em russo e contém a
pontuação obtida pelo estudante.

Um problema muito interessante que já caiu no Torneio é o seguinte:

(a) Duas pessoas realizam um truque. A primeira retira 5 cartas de um
baralho de 52 cartas (previamente embaralhado por um membro da platéia),
olha-as,  e coloca-as em uma linha da esquerda para a direita: uma com a
face para baixo (não necessariamente a primeira),  e a outras com a face
para cima. A segunda pessoa deve adivinhar a carta que esté com a face para
baixo. Prove que elas podem combinar um sistema que sempre torna isto
possível.

(b) Prove que as pessoas ainda podem realizar o truque se a carta oculta for
colocado no bolso da 1a pessoa.


Eu sei resolver o ítem (a), mas não o (b).

Paulo José







Re: 2 problemas..

2001-10-18 Por tôpico Ralph Teixeira

Hmmm...

O primeiro nao pode ser verdade Afinal, a=0, c=2 e b=o que quer que
precise satisfaz a primeira parte mas nao a segunda. Serah que a,b e c nao
eram naturais?

Se forem naturais... bom, ainda nao consegui fazer para |a/2|+|c/2| nem
achar um contra-exemplo. Se fosse |a/2|+|b/2|, eu saberia fazer: analise
tudo a modulo 8. Os restos de n^2 sao sempre 0, 1 ou 4 modulo 8 (se n eh
natural). Assim, para que a^2+b^2+1=c^2, devemos ter

- a^2=b^2=0 (modulo 8) e c^2=1 (modulo 8)
OU
- a^2=b^2=4 e c^2=1 (tudo modulo 8).

No primeiro caso, a e b sao divisiveis por 4 e entao a/2 e b/2 sao
pares. No segundo, a e b deixam resto 2 na divisao por 4, e entao a/2 e b/2
sao impares e acabou.

Depois eu vou pensar no |a/2|+|c/2|

Abraco,
Ralph

- Original Message -
From: Carlos Stein Naves de Brito [EMAIL PROTECTED]
To: [EMAIL PROTECTED]
Sent: Wednesday, October 17, 2001 7:39 PM
Subject: 2 problemas..


Gostaria de ver soluções para esses probleminhas  que estão me entalando.
Valeu.
1-Sejam a, b e c reais tais que a^2 + b^2 +1 = c^2. Prove que |a/2| + |c/2|
é par. |x| é a parte inteira de x.
2-Seja g(x)=ax^2 + bx + c uma função quadrática com coeficientes reais(a não
nulo) tal que a equação g(g(x)) = x tem quatro raízes reais distintas.
Demontre que não existe nenhuma função f:R-R tal que f(f(x)) = g(x) para
todo x real.





<    3   4   5   6   7   8